LSAT and Law School Admissions Forum

Get expert LSAT preparation and law school admissions advice from PowerScore Test Preparation.

User avatar
 Dave Killoran
PowerScore Staff
  • PowerScore Staff
  • Posts: 5849
  • Joined: Mar 25, 2011
|
#27195
Complete Question Explanation
(The complete setup for this game can be found here: lsat/viewtopic.php?t=11436)

The correct answer choice is (B)


The condition in the question stem leads to the following two basic scenarios, based on whether F or G is first:
June 03_M12_game#2_L7_explanations_game#2_#10_diagram_1.png
In scenario #1, K cannot be selected because F is first (this inference was discussed in the game setup: when K is selected, G must be first). Thus, only G, H, and M remain to fill the final two spaces. If both G and H are selected, then H must rank better than G. If M is selected, then H must be selected.

In scenario #2, neither H nor M can be selected: M cannot be selected with G, and H cannot be selected when G is first (when H is selected, F must be first). Thus, only K and F remain to fill the final two spaces. And because of the fifth rule, K must rank ahead of F.

The information above yields the following completed scenarios:
June 03_M12_game#2_L7_explanations_game#2_#10_diagram_2.png
Consequently, answer choice (B) is correct.
 rachelgstevens
  • Posts: 4
  • Joined: Jan 22, 2019
|
#63148
Is this one of those questions where the answer choice cannot be true even without the local rule? I didn't draw templates/scenarios because I had one of the inferences as K :arrow: G1, which would mean that K is never ranked higher than G.
User avatar
 Dave Killoran
PowerScore Staff
  • PowerScore Staff
  • Posts: 5849
  • Joined: Mar 25, 2011
|
#63151
rachelgstevens wrote:Is this one of those questions where the answer choice cannot be true even without the local rule? I didn't draw templates/scenarios because I had one of the inferences as K :arrow: G1, which would mean that K is never ranked higher than G.
Yes, very good!
 g_lawyered
  • Posts: 211
  • Joined: Sep 14, 2020
|
#89080
Hi P.S.,
I didn't recognize/ make the Limited Solution Set this question uses to answer. I also didn't see it in the Setup/Diagram explanation section either. Can this game be solved completely based on these 2 Solution sets only? Or was I correct in not making Solution sets since most of the rules are conditional statements? I'd appreciate any feedback because I'm taking long to answer these Local Questions without having Solution sets.

Thanks in advance
 g_lawyered
  • Posts: 211
  • Joined: Sep 14, 2020
|
#89081
As a follow up to my last post,
Is it correct to infer that ONLY F or G could be 1st (I didn't recognize the 2 Solution Sets of having either F or G 1st only). I had some trouble understanding this rule because I understood that other variables could go 1st. For example, in answering Question #7 I tested answer choice C. I came up with Solution set in this order: I, L, K, F/H, H/F (G and M are out). Is it incorrect to say that I COULD go 1st?
Thanks in advance
 Rachael Wilkenfeld
PowerScore Staff
  • PowerScore Staff
  • Posts: 1358
  • Joined: Dec 15, 2011
|
#89394
Hi GGIBA003

This question uses two possible sets because that's all that can happen here. We know that either F or G has to go first because of the rules. The rule right after L ranks second is that either F or G is first. That's a global rule, and therefore we always have to have one of those two first. For this specific question, we know even more---I couldn't be first in this one anyway because I has to be third in this local question.

We want to draw out what we know could happen based on both the main diagram and the new local rule. When creating this mini diagram set up, I started with putting L 2 and I 3. There were then two options for how the game could go. Either F or G has to go first, and so we make two different diagrams, one for each F or G going first.

Whenever we think about making templates/possibilities, either in a local question or a main game, I like to ask myself "will these templates/possibilities help me see something larger about the relationships?" Here, the answer is absolutely. With putting either F/G first, the L and the I, our mini diagrams have 3 of the 5 set before we even have to make any inferences. That is something I expect to yield helpful information, and in fact, as we see from the first post in this thread, it does.

Hope that helps!

Get the most out of your LSAT Prep Plus subscription.

Analyze and track your performance with our Testing and Analytics Package.